Homework 4 Solutions 26.5. Let ∑ N=1 an and ∑ N=1 Bn Be Absolutely

Homework 4 Solutions 26.5. Let ∑ N=1 an and ∑ N=1 Bn Be Absolutely

Homework 4 Solutions Math 171, Spring 2010 Please send corrections to [email protected] P1 P1 P1 p 26.5. Let n=1 an and n=1 bn be absolutely convergent series. Prove that the series n=1 janbnj converges. P1 P1 P1 Solution. Since n=1 janj and n=1 jbnj converge, by Theorem 23.1, n=1 janj + jbnj converges. Note that for all n, we have p p p janbnj = janjjbnj ≤ (janj + jbnj)(janj + jbnj) = janj + jbnj: P1 So by Theorem 26.3 (Comparison Test), since n=1 janj + jbnj converges absolutely, we see that P1 p n=1 janbnj converges absolutely, and hence converges as all terms are positive. 27.1. Find the radius of convergence R of each of the power series. Discuss the convergence of the power series at the points jx − tj = R. Solution. P1 x2n−1 (a) n=1 (2n−1)! Note that x2(n+1)−1 x2 lim (2(n+1)−1)! = lim = 0 n!1 x2n−1 n!1 (2n + 1)(2n) (2n−1)! so by the ratio test, the series converges absolutely for all x, and so R = 1. P1 n(x−1)n (b) n=1 2n Note that (n+1)(x−1)(n+1) (n+1) n + 1 x − 1 x − 1 lim 2 = lim = n!1 n(x−1)n n!1 n 2 2 2n so by the ratio test, the series converges absolutely if jx − 1j < 2 and diverges if jx − 1j > 2. Thus P1 P1 n R = 2. If jx − 1j = 2 then the power series diverges, since n=1 n and n=1(−1) n diverge. P1 n!(x+2)n (c) n=1 2n Note that (n+1)!(x+2)(n+1) ( (n+1) (n + 1)(x + 2) 0 if x = −2 lim 2 = lim = n!1 n!(x+2)n n!1 2 1 otherwise 2n so by the ratio test, the series diverges if jx + 2j > 0. Thus R = 0. If jx + 2j = 0 then the power series converges by the ratio test, or by the comment after Definition 27.1. P1 1=n n (d) n=1(n − 1)x By Theorems 16.7 and 25.1 (Alternating Series Test), the series converges when x = −1. 1 n By equality (16.3) in the proof of Theorem 16.7, we know (1 + n ) ≤ n. Taking n-th roots of 1 1=n P1 1 both sides and subtracting 1, we get n ≤ n − 1. We know that the series n=1 n diverges, hence P1 1=n so does n=1 n − 1. Thus the series diverges when x = 1. 1 Hence by Theorem 27.2, we have R = 1, and we have already discussed the behavior of the power series when jxj = 1. P1 (x−π)n (e) n=2 n(n−1) Note that (x−π)(n+1) n − 1 lim (n+1)n = lim (x − π) = jx − πj n!1 (x−π)n n!1 n + 1 n(n−1) so by the ratio test, the series converges absolutely if jx − πj < 1 and diverges if jx − πj > 1. Thus P1 1 P1 (−1)n R = 1. If jx−πj = 1 then the power series converges, since n=1 n(n−1) and n=1 n(n−1) converge. P1 n 27.2. Suppose that the power series n=0 an(x − t) has radius of convergence R. Let p be an integer. P1 p n Prove that the power series n=0 n an(x − t) has radius of convergence R. Solution. We consider thhree cases. 1=n 1 First case: 0 < R < 1. By Theorem 27.2, we have lim supn!1 janj = R . Note that p=n 1=n p p limn!1 n = (limn!1 n ) = 1 = 1 by Exercise 17.4 and Theorem 16.7. Note also that 1=n 1=n 1 fjanj g is a bounded sequence as lim supn!1 janj = R < 1. Hence Theorem 20.8 applies, and we get p 1=n p=n 1=n p=n 1=n 1 lim sup(n janj) = lim sup n janj = lim n lim sup janj = n!1 n!1 n!1 n!1 R P1 p n So n=0 n an(x − t) has radius of convergence R. 1 Second case: R = 1. The method in the case above works, after replacing R with zero. 1=n Third case: R = 0. Hence by Definition 21.2 the sequence fjanj g is not bounded above. Let K p=n p=n 1 be a positive number. Since limn!1 n = 1, there exists some N such that n ≥ 2 for all n ≥ N. p 1=n p 1=M Since fjn anj g is not bounded above, there exists some M > N such that jM aM j > 2K (for p 1=1 p 1=N p 1=n otherwise maxfj1 a1j ; ··· ; jN aN j ; 2Kg would be an upper bound for fjn anj g). Hence p 1=M p=M p 1=M 1 p 1=n jM aM j = M jM aM j > 2 2K = K. This shows that fjn anj g is not bounded above, P1 p n so n=0 n an(x − t) has radius of convergence 0. 28.1. Prove that the following series converge conditionally. Solution. (a) 1 + p1 − p2 + p1 + p1 − p2 + ··· 2 3 4 5 6 p1 Let fang be the sequence 1; 1; −2; 1; 1; −2;::: . Let bn = n . Note that the sequence of partial P1 sums of n=1 an is bounded and fbng is a decreasing sequence with limit 0. We apply Corollary 28.3 (Dirichlet's Test) to see that 1 1 2 1 1 2 X 1 + p − p + p + p − p + ··· = anbn 2 3 4 5 6 n=1 converges. P1 n (1−n)=n (b) n=1(−1) n n −1 1=n P1 Let an = (−1) n and let bn = n . Note that n=1 an converges by Theorem 25.1 (Alternat- ing Series Test) and that fbng is a bounded monotone sequence for n sufficiently large by Theorem 16.7. We apply Corollary 28.6 (Abel's Test) to see that 1 1 1 X n (1−n)=n X n −1 1=n X (−1) n = (−1) n n = anbn n=1 n=1 n=1 converges. 2 P1 28.3. Give an example of a convergent series n=1 an and a positive sequence fbng such that limn!1 bn = P1 0 and n=1 anbn diverges. ( (−1)n 0 n odd Solution. Let an = p and bn = . n p1 n n even P1 Then n=1 an converges by Theorem 25.1 (Alternating Series Test) and limn!1 bn = 0. Note P1 P1 1 n=1 anbn = m=1 2m diverges by Corollary 24.3. Pp 28.6. Let fang be a periodic sequence, an = an+p for all n and for fixed p, for which n=1 an = 0. Let P1 fbng be a decreasing sequence with limit 0. Prove that n=1 anbn converges. P1 Pp Solution. Consider the partial sums sn of the series n=1 an. Since fang is periodic and n=1 an = Pp 0, we see that sn+p = sn + i=1 an+i = sn for all n. Hence sn 2 fs1; :::; spg for all n. This shows P1 that the sequence of partial sums of the series n=1 an is bounded. Hence Corollary 28.3 (Dirichlet's P1 Test) applies, and we conclude n=1 anbn converges. P1 xn 29.3. Prove that n=0 n! converges absolutely for every x and 1 1 1 X xn X yn X (x + y)n ( )( ) = n! n! n! n=0 n=0 n=0 Solution. Note that xn+1 (n+1)! x lim n = lim = 0 n!1 x n!1 n! n + 1 so by the ratio test, the series converges absolutely for every x. xn yn By letting an = n! and bn = n! in Theorem 29.9, we get that 1 1 1 1 X xn X yn X X ( )( ) = ( a )( b ) n! n! n n n=0 n=0 n=0 n=0 1 X = cn n=0 1 n X X = ( aibn−i) n=0 i=0 1 n X X xi yn−i = ( ) i! (n − i)! n=0 i=0 1 n X 1 X n = ( xiyn−i) n! i n=0 i=0 1 X (x + y)n = n! n=0 where the last equality is by the binomial theorem. P1 P1 29.5. Give an example of convergent series n=0 an and n=0 bn whose Cauchy product diverges. Solution. 3 1 1 1 Let P a = P b = P (−1)n p 1 . This series converges by Theorem 25.1 (Alternating n=0 n n=0 n n=0 n+1 Series Test). The n-th term in the Cauchy product is n n n X X k n−k 1 1 n X 1 1 cn = akbn−k = (−1) (−1) p p = (−1) p p k + 1 n + k + 1 k + 1 n − k + 1 k=0 k=0 k=0 Since p 1 ≥ p 1 and likewise p 1 ≥ p 1 , we have k+1 n+1 n−k+1 n+1 n X 1 1 jc j ≥ = (n + 1) = 1 n n + 1 n + 1 k=0 P1 So limn!1 cn 6= 0, so the series n=0 cn diverges by Theorem 22.3. 4.

View Full Text

Details

  • File Type
    pdf
  • Upload Time
    -
  • Content Languages
    English
  • Upload User
    Anonymous/Not logged-in
  • File Pages
    4 Page
  • File Size
    -

Download

Channel Download Status
Express Download Enable

Copyright

We respect the copyrights and intellectual property rights of all users. All uploaded documents are either original works of the uploader or authorized works of the rightful owners.

  • Not to be reproduced or distributed without explicit permission.
  • Not used for commercial purposes outside of approved use cases.
  • Not used to infringe on the rights of the original creators.
  • If you believe any content infringes your copyright, please contact us immediately.

Support

For help with questions, suggestions, or problems, please contact us